- PowerScore Staff
- Posts: 5972
- Joined: Mar 25, 2011
- Wed Jul 13, 2016 12:10 pm
#27097
Complete Question Explanation
(The complete setup for this game can be found here: lsat/viewtopic.php?f=325&t=6210)
The correct answer choice is (B)
The condition in the question stem establishes the following setup:
The question then proceeds to ask for the variable that could be replaced by K without creating a violation.
Answer choice (A) is incorrect because K cannot substitute for any member of the planting committee; if K is a member without the presence of J, the third rule would be violated.
Answer choice (B) is the correct answer choice. Removing G from the trails committee and replacing it with K would not cause any violations.
Answer choice (C) is incorrect because from the last rule the two committees must have at least one member in common, and H is the only member in common.
Answer choice (D) is incorrect because K cannot substitute for any member of the planting committee; if K is a member without the presence of J, the third rule would be violated.
Answer choice (E) is incorrect because K cannot substitute for any member of the planting committee; if K is a member without the presence of J, the third rule would be violated. In addition, M must appear on at least one of the committees and so removing M would violate the fourth rule.
(The complete setup for this game can be found here: lsat/viewtopic.php?f=325&t=6210)
The correct answer choice is (B)
The condition in the question stem establishes the following setup:
The question then proceeds to ask for the variable that could be replaced by K without creating a violation.
Answer choice (A) is incorrect because K cannot substitute for any member of the planting committee; if K is a member without the presence of J, the third rule would be violated.
Answer choice (B) is the correct answer choice. Removing G from the trails committee and replacing it with K would not cause any violations.
Answer choice (C) is incorrect because from the last rule the two committees must have at least one member in common, and H is the only member in common.
Answer choice (D) is incorrect because K cannot substitute for any member of the planting committee; if K is a member without the presence of J, the third rule would be violated.
Answer choice (E) is incorrect because K cannot substitute for any member of the planting committee; if K is a member without the presence of J, the third rule would be violated. In addition, M must appear on at least one of the committees and so removing M would violate the fourth rule.
You do not have the required permissions to view the files attached to this post.
Dave Killoran
PowerScore Test Preparation
Follow me on X/Twitter at http://twitter.com/DaveKilloran
My LSAT Articles: http://blog.powerscore.com/lsat/author/dave-killoran
PowerScore Podcast: http://www.powerscore.com/lsat/podcast/
PowerScore Test Preparation
Follow me on X/Twitter at http://twitter.com/DaveKilloran
My LSAT Articles: http://blog.powerscore.com/lsat/author/dave-killoran
PowerScore Podcast: http://www.powerscore.com/lsat/podcast/